Prep Test 63 Logic question (June 2011) Forum

Prepare for the LSAT or discuss it with others in this forum.
Post Reply
User avatar
PLXTDNR

Bronze
Posts: 155
Joined: Thu Mar 01, 2012 12:59 pm

Prep Test 63 Logic question (June 2011)

Post by PLXTDNR » Thu May 31, 2012 12:33 pm

This question almost drove me insane. I'd like to hear some thoughts on what you think the answer is, and most importantly WHY.

Question 22, Section 3

Principle: A police officer is eligible for a Mayor's Commendation if the office has an exemplary record, but not otherwise; an officer eligible for the award who did something this year that exceeded what could be reasonably expected of a police officer should receive the award if the act saved someone's life.

Conclusion: Officer Franklin should receive a Mayor's Commendation but Officer Penn should not.

From which one of the following facts can the conclusion be properly drawn using the principle?

A. In saving a child from drowning this year, Franklin and Penn both risked their lives beyond what could be reasonably expected of a police officer. Franklin has an exemplary record but Penn does not.

B. Both Franklin and Penn have exemplary records, and each officer saved a child from drowning earlier this year. However, in doing so, Franklin went beyond what could be reasonably expected of a police officer; Penn did not.

I wont' bother putting CDE in here, because one of these two is the right answer and I for the life of me cannot figure out which it is, and I've gotten 3 different takes on why the correct answer is right.

Thanks!

User avatar
flem

Diamond
Posts: 12882
Joined: Mon Oct 10, 2011 6:44 pm

Re: Prep Test 63 Logic question (June 2011)

Post by flem » Thu May 31, 2012 12:45 pm

JFC that's ugly, but it seems A is the better answer to me. It seems that doing the same act while one has an exemplary record and one didn't would clearly make one ineligible. Which one is correct?

Edit: A also matched my prephrase.

User avatar
PLXTDNR

Bronze
Posts: 155
Joined: Thu Mar 01, 2012 12:59 pm

Re: Prep Test 63 Logic question (June 2011)

Post by PLXTDNR » Thu May 31, 2012 12:50 pm

tfleming09 wrote:JFC that's ugly, but it seems A is the better answer to me. It seems that doing the same act while one has an exemplary record and one didn't would clearly make one ineligible. Which one is correct?
A is correct; but they both SEEM to be correct - one of the responses I got as to why "A" is correct was this:

A specified that BOTH officers worked the same drowning, both risking their lives beyond etc...but one is ineligible due to a non-exemplary record.

In B, BOTH have exemplary records, but worked two different drownings where Franklin went above and beyond and Penn did not. BUT - according to this individual - we have no way of knowing if some OTHER act committed by Penn should have garned him the commendation (ie. the next day he went into a burning building b/c the firemen were still too far away and saved 3 children and a cat).

I personally think whoever wrote that question should be summarily executed.

User avatar
flem

Diamond
Posts: 12882
Joined: Mon Oct 10, 2011 6:44 pm

Re: Prep Test 63 Logic question (June 2011)

Post by flem » Thu May 31, 2012 12:51 pm

PLXTDNR wrote: A is correct; but they both SEEM to be correct - one of the responses I got as to why "A" is correct was this:

A specified that BOTH officers worked the same drowning, both risking their lives beyond etc...but one is ineligible due to a non-exemplary record.

In B, BOTH have exemplary records, but worked two different drownings where Franklin went above and beyond and Penn did not. BUT - according to this individual - we have no way of knowing if some OTHER act committed by Penn should have garned him the commendation (ie. the next day he went into a burning building b/c the firemen were still too far away and saved 3 children and a cat).

I personally think whoever wrote that question should be summarily executed.
1) I think that's a fairly cogent assessment

2) lol'd at the bolded

Just remember principle questions are super annoying and the question stem is always "which one of the following most closely adheres" and A is the clearer, stronger answer.

hopefully shinners or someone can give a better analysis than me.

User avatar
hereyago

Bronze
Posts: 103
Joined: Fri Jun 17, 2011 5:02 pm

Re: Prep Test 63 Logic question (June 2011)

Post by hereyago » Thu May 31, 2012 1:01 pm

A is correct because Frank is eligible. Penn is not. It is pretty clear and I think you understand it as well so I won't go over it.

I'm guessing the reason it is driving you insane is because B looks correct as well. In B, Frank is clearly eligible. He has an exemp rec, and he did something this year that exceeded what a cop should do + that act saved the kid's life. However, there is an issue with Penn. We actually AREN'T 100% SURE if he is ineligible or not. Why? Because we don't know if he has performed another act that was beyond what a cop should do + the act saved another's life. The conclusion clearly states that Penn is not eligible (or should not receive but whatever. same crap).

If B had said that they both had exemp recs, Frank performed that act described in B, and Penn did not perform any other acts that saved someone's life, then it would be correct.


edit: oh you answered it yourself above. did not see your 2nd post.

Want to continue reading?

Register now to search topics and post comments!

Absolutely FREE!


User avatar
PLXTDNR

Bronze
Posts: 155
Joined: Thu Mar 01, 2012 12:59 pm

Re: Prep Test 63 Logic question (June 2011)

Post by PLXTDNR » Thu May 31, 2012 1:15 pm

tfleming09 wrote:Just remember principle questions are super annoying and the question stem is always "which one of the following most closely adheres" and A is the clearer, stronger answer.

hopefully shinners or someone can give a better analysis than me.
Yes, I seem to have a problem with things that are clearer and stronger. I want to be a lawyer, therefore I like it muddy ;)

User avatar
PLXTDNR

Bronze
Posts: 155
Joined: Thu Mar 01, 2012 12:59 pm

Re: Prep Test 63 Logic question (June 2011)

Post by PLXTDNR » Thu May 31, 2012 1:16 pm

hereyago wrote:A is correct because Frank is eligible. Penn is not. It is pretty clear and I think you understand it as well so I won't go over it.

I'm guessing the reason it is driving you insane is because B looks correct as well. In B, Frank is clearly eligible. He has an exemp rec, and he did something this year that exceeded what a cop should do + that act saved the kid's life. However, there is an issue with Penn. We actually AREN'T 100% SURE if he is ineligible or not. Why? Because we don't know if he has performed another act that was beyond what a cop should do + the act saved another's life. The conclusion clearly states that Penn is not eligible (or should not receive but whatever. same crap).

If B had said that they both had exemp recs, Frank performed that act described in B, and Penn did not perform any other acts that saved someone's life, then it would be correct.


edit: oh you answered it yourself above. did not see your 2nd post.

Thanks - yep, I agree - if I treated LR like games, I would have chosen A and moved on - but I've been taught the "cleanest dirty shirt" method, which means I read all answers even if I think I've found the right one in A. I might have to rethink that strategy on my next PT and see how I do just not second guessing myself.

TERS

Bronze
Posts: 161
Joined: Thu May 24, 2012 11:29 pm

Re: Prep Test 63 Logic question (June 2011)

Post by TERS » Thu May 31, 2012 2:13 pm

Edit: I'm dumb
Last edited by TERS on Thu May 31, 2012 2:30 pm, edited 1 time in total.

User avatar
flem

Diamond
Posts: 12882
Joined: Mon Oct 10, 2011 6:44 pm

Re: Prep Test 63 Logic question (June 2011)

Post by flem » Thu May 31, 2012 2:20 pm

I think "office" was supposed to be "officer" and that was a typo bro

Want to continue reading?

Register for access!

Did I mention it was FREE ?


TERS

Bronze
Posts: 161
Joined: Thu May 24, 2012 11:29 pm

Re: Prep Test 63 Logic question (June 2011)

Post by TERS » Thu May 31, 2012 2:26 pm

tfleming09 wrote:I think "office" was supposed to be "officer" and that was a typo bro
LOL! Well that was a waste of time! I think I'll look up the test next time!

User avatar
PLXTDNR

Bronze
Posts: 155
Joined: Thu Mar 01, 2012 12:59 pm

Re: Prep Test 63 Logic question (June 2011)

Post by PLXTDNR » Thu May 31, 2012 2:39 pm

TERS wrote:
tfleming09 wrote:I think "office" was supposed to be "officer" and that was a typo bro
LOL! Well that was a waste of time! I think I'll look up the test next time!
My key doesn't wok vey well.

User avatar
LexLeon

Bronze
Posts: 397
Joined: Fri Oct 07, 2011 11:03 pm

Re: Prep Test 63 Logic question (June 2011)

Post by LexLeon » Sun Jun 03, 2012 9:18 pm

The writer really threw us a bone by making (A) correct here. I was kind of pressed for time on this section, read (A) and believed it was correct, selected it, and moved on.

Fortunately, it was correct. Unfortunately, I was not concentrating hard enough to note the biconditional relationship between eligibility and having an exemplary record at that time. Once that is clear, (A) is too:

It meets only two of the three jointly sufficient conditions for the Award for Penn--where the missing condition is in fact also necessary to the award--and all three of these for Franklin. This by logical necessity rules out Penn.

(B) through (E) can really drain time and energy; this is why I laud the writer for CR'ing (A). (B), especially so, is such a drain, for it turns solely on (as far as I can see) the subtlety regarding the possibility that Penn could have at a different time met that third necessary and sufficient condition.

If someone sees something else that is wrong with it, please send me a message.

User avatar
jhw219

New
Posts: 77
Joined: Sun Dec 11, 2011 12:44 am

Re: Prep Test 63 Logic question (June 2011)

Post by jhw219 » Mon Jun 04, 2012 12:44 am

I had my friend who's super into logic (and never seen an LSAT question) explain this one to me and I feel that his answer sounds right, but feel free to correct me if i'm wrong!

I think the key is to do it as a formal logic question.

There's the first premise of if exemplary, then eligible but also if not exemplary, then not eligible. This part is obviously not really a factor in answer B cuz they both have exemplary records.

The second part of the premise is where I think it is useful to use formal logic.
if eligible AND exceeded what was expected AND saved life -> should get award

for answer B, Franklin has all three but Penn is missing the "exceeded what was expected" and as we know (taking out the other two factors) exceeded what was expected -> should get award does NOT equal ~exceeded what was expected -> ~should get award.

does this look right?

Register now!

Resources to assist law school applicants, students & graduates.

It's still FREE!


User avatar
PLXTDNR

Bronze
Posts: 155
Joined: Thu Mar 01, 2012 12:59 pm

Re: Prep Test 63 Logic question (June 2011)

Post by PLXTDNR » Fri Jun 08, 2012 9:29 am

jhw219 wrote:I had my friend who's super into logic (and never seen an LSAT question) explain this one to me and I feel that his answer sounds right, but feel free to correct me if i'm wrong!

I think the key is to do it as a formal logic question.

There's the first premise of if exemplary, then eligible but also if not exemplary, then not eligible. This part is obviously not really a factor in answer B cuz they both have exemplary records.

The second part of the premise is where I think it is useful to use formal logic.
if eligible AND exceeded what was expected AND saved life -> should get award

for answer B, Franklin has all three but Penn is missing the "exceeded what was expected" and as we know (taking out the other two factors) exceeded what was expected -> should get award does NOT equal ~exceeded what was expected -> ~should get award.

does this look right?
It sounds like you're saying that B should be the right answer, but it's not (I did choose b - my understanding of the 'wrongness' of b is not that it turns on the 'exceeded what was expected,' but that there's a subtle nuance of IN THAT INSTANCE ONLY he did not exceed what was expected. It does not state that at another time he did or did not exceed what was expected. It was a horrible joke by the author, but I guess, as another commenter pointed out, he did a favor by making "A" the answer (if one were smart enough to realize that it met all the conditions and just move on LOL!)

Taus11

New
Posts: 65
Joined: Tue Jan 03, 2012 5:05 am

Re: Prep Test 63 Logic question (June 2011)

Post by Taus11 » Fri Jun 08, 2012 11:29 am

It's not even a subtle nuance. You are finding an answer that best supports why Penn shouldn't get the award. If he does not have the record he clearly should not, as the premise indicates. (B) tells us literally nothing about what Penn should or should not receive. It's not implied, it's just textual.

User avatar
PLXTDNR

Bronze
Posts: 155
Joined: Thu Mar 01, 2012 12:59 pm

Re: Prep Test 63 Logic question (June 2011)

Post by PLXTDNR » Fri Jun 08, 2012 3:48 pm

Taus11 wrote:It's not even a subtle nuance. You are finding an answer that best supports why Penn shouldn't get the award. If he does not have the record he clearly should not, as the premise indicates. (B) tells us literally nothing about what Penn should or should not receive. It's not implied, it's just textual.
You're way more of a nuance bloodhound than I am. For me, it was quite subtle (I know very few people who got the nuance quickly).

User avatar
nsideirish

Bronze
Posts: 411
Joined: Sun Dec 13, 2009 10:32 am

Re: Prep Test 63 Logic question (June 2011)

Post by nsideirish » Fri Jun 08, 2012 4:04 pm

This may be a terrible way of phrasing it, but B doesn't "close the gap". Just because Penn didn't satisfy that one condition to receive a medal does not mean that there was something else he could have done to receive one. Doing something beyond expectations that saves someone's life (as long as you have an exemplary record) is sufficient for receiving a medal. But just because you haven't fulfilled that sufficient element does not mean you should not receive an award.

Put simply, Penn could have done something else that means he should receive the award.

Hope that makes sense.

Get unlimited access to all forums and topics

Register now!

I'm pretty sure I told you it's FREE...


Post Reply

Return to “LSAT Prep and Discussion Forum”